Đến nội dung

Hoang Nhat Tuan nội dung

Có 1000 mục bởi Hoang Nhat Tuan (Tìm giới hạn từ 25-05-2020)



Sắp theo                Sắp xếp  

#598380 Cho các số thực không âm $x,y,z$ thỏa $\sqrt{1+x^2...

Đã gửi bởi Hoang Nhat Tuan on 15-11-2015 - 06:48 trong Bất đẳng thức và cực trị

 

Cho các số thực không âm $x,y,z$ thỏa $\sqrt{1+x^2}+\sqrt{1+2y}+\sqrt{1+2z}=5$.Chứng minh rằng:
$$ 2x^3+y^3+z^3 \le 64$$

 

Trước tiên dự đoán điểm rơi: $x=z=0;y=4$ hoặc $x=y=0;z=4$.

Do đó ta sử dụng đánh giá $P=2x^3+y^3+z^3\leq 2x^3+(y+z)^3$

Bổ đề: $\sqrt{1+x}+\sqrt{1+y}\geq 1+\sqrt{1+x+y}<=>\sqrt{(1+x)(1+y)}\geq \sqrt{1+x+y}$ (luôn đúng).

Áp dụng vào giả thiết ta thu được:

$5=\sqrt{1+2y}+\sqrt{1+2z}+\sqrt{1+x^2}\geq 1+\sqrt{1+2(y+z)}+\sqrt{1+x^2}\geq 2+\sqrt{1+2(y+z)+x^2}<=>8\geq x^2+2(y+z)$

Do đó: $y+z\leq 4-\frac{x^2}{2}$

Mà $y,z\geq 0$ do đó $\frac{x^2}{2}\leq 4$ hay $x\leq 2\sqrt{2}$

Và $P\leq 2x^3+(4-\frac{x^2}{2})^3$

Giờ chỉ cần khảo sát hàm trên $\left [ 0;2\sqrt2 \right ]$ nữa là ra :)




#597958 Chứng minh IQ vuông góc BC

Đã gửi bởi Hoang Nhat Tuan on 12-11-2015 - 09:37 trong Hình học phẳng

Đề bài: Cho tam giác ABC, trung tuyến AM, phân giác AN (M,N thuộc BC). Từ điểm N kẻ đường vuông góc với NA cắt AB,AM lần lượt tại P,Q. Qua P kẻ đường vuông góc với AP cắt AN tại I. Chứng minh IQ vuông góc BC.

Lời giải: (Ai có cách đơn giản hơn cho em xin ạ :) )

Hình gửi kèm

  • 12219439_557755327710486_5016378512487553711_n.jpg
  • 12244486_557755291043823_8081673356054902984_o.jpg



#597704 Chứng minh : M,G,K thẳng hàng

Đã gửi bởi Hoang Nhat Tuan on 10-11-2015 - 18:48 trong Hình học

Vẽ đường kính $AJ$

Khi đó dễ chứng minh $HCJB$ là hình bình hành.

$I$ là trung điểm $BC$ nên cũng là trung điểm $HJ$

Mặt khác dễ chứng minh $IH=IG$ từ đó suy ra $OI$ là đường trung trực của $GJ$

Suy ra $\widehat{OGI}=\widehat{OJI}=\widehat{AJK}=\widehat{AGK}$  (1)

Lại có: $OG^2=OC^2=OI.OM$ nên $\Delta OGI\sim \Delta OMG=>\widehat{OGI}=\widehat{OMG}$  (2)

Từ (1) và (2) kết hợp với $AG//OM$ suy ra điều phải chứng minh.

 

Hình gửi kèm

  • Untitled.png



#597699 Chứng minh $P,J,O$ thẳng hàng.

Đã gửi bởi Hoang Nhat Tuan on 10-11-2015 - 17:51 trong Hình học phẳng

Cho tam giác $ABC$, một điểm $P$ thuộc tam giác sao cho $\widehat{ABP}=\widehat{ACP}$, trên $AB,AC$ lấy $E,F$ sao cho $PE=PB;PF=PC$. Gọi $J,O$ làm tâm đường tròn ngoại tiếp tam giác $PEF$ và $ABC$. Chứng minh $P,J,O$ thẳng hàng.

Hình gửi kèm

  • Untitled.png



#597698 Chứng minh : M,G,K thẳng hàng

Đã gửi bởi Hoang Nhat Tuan on 10-11-2015 - 17:47 trong Hình học

Bài toán

Cho tam giác ABC nhọn (AB<AC) nội tiếp đường tròn (O).Ba đường cao BD,CE,AF cắt nhau tại H. Chứng minh : 

a.Tứ giác BEDC nội tiếp . Xác định tâm I của đường tròn ngoại tiếp tứ giác này .

b.Tiếp tuyến tại B và C của (O) cắt nhau tại M. Tia IH cắt (O) tại K. Chứng minh : M,G,K thẳng hàng

Giúp em câu b thôi :D

G là điểm nào vậy bạn @@




#597468 Chứng minh $IG$ vuông góc với $EF$

Đã gửi bởi Hoang Nhat Tuan on 08-11-2015 - 23:57 trong Hình học phẳng

Cho tam giác nhọn $ABC$ với $BC$ là cạnh nhỏ nhất. Đường tròn nội tiếp $(I)$ của tam giác tiếp xúc với $BC, CA, AB$ theo thứ tự tại $X, Y, Z$. Gọi $G$ là trọng tâm tam giác $XYZ$. Trên các tia $BA, CA$ lấy các điểm $E, F$ sao cho $BE = CF = BC$. Chứng minh $IG$ vuông góc với $EF$

Bài này dùng định lý 4 điểm kết hợp với đường thẳng Ơ-le là ra :)




#597447 $\sum \frac{1}{a+b}+\frac{1...

Đã gửi bởi Hoang Nhat Tuan on 08-11-2015 - 22:02 trong Bất đẳng thức và cực trị

Cho a,b,c > 0, chứng minh rằng:
$\sum \frac{1}{a+b}+\frac{1}{2\sqrt[3]{abc}}\geq \frac{(a+b+c+\sqrt[3]{abc})^{2}}{(a+b)(b+c)(c+a)}$

Ta có: $\sum \frac{1}{a+b}+\frac{1}{2\sqrt[3]{abc}}$

$=\sum \frac{c^2}{c^2(a+b)}+\frac{\sqrt[3]{a^2b^2c^2}}{2abc}\geq \frac{(a+b+c+\sqrt[3]{abc})^2}{\sum c^2(a+b)+2abc}=\frac{(a+b+c+\sqrt[3]{abc})^2}{(a+b)(b+c)(c+a)}$




#597386 Chứng minh rằng $(a+b)^3\geq 4ab\sqrt[2]{a^2+b^2}...

Đã gửi bởi Hoang Nhat Tuan on 08-11-2015 - 15:49 trong Bất đẳng thức và cực trị

bài 1. Chứng minh rằng 

 

$(a+b)^3\geq 4ab\sqrt[2]{a^2+b^2}$

 

bài 2

 

CMR:    $(\frac{a}{b^2}+\frac{b}{a^2})(\frac{a^2}{b^3}+\frac{b^3}{a^3})\geqslant4$  với a,b>0 và a+b=2

Bài 1:

$(a+b)^3=(a+b)(a^2+2ab+b^2)\geq 2\sqrt{ab}.2\sqrt{a^2+b^2}.\sqrt{2ab}=4\sqrt{2}.ab.\sqrt{a^2+b^2}$

Bài 2:

$\frac{a}{b^2}+\frac{b}{a^2}+a+b\geq 2(\frac{a}{b}+\frac{b}{a})\geq 4=>\frac{a}{b^2}+\frac{b}{a^2}\geq 2$

$\frac{a^2}{b^3}+b+\frac{b^2}{a^3}+a\geq 2(\frac{a}{b}+\frac{b}{a})\geq 4=>\frac{a^2}{b^3}+\frac{b^2}{a^3}\geq 2$

Từ đó suy ra điều phải chứng minh. :)




#597338 ĐỀ THI CHỌN HSG CẤP TỈNH LỚP 12 TỈNH LONG AN 2015-2016

Đã gửi bởi Hoang Nhat Tuan on 08-11-2015 - 10:04 trong Thi HSG cấp Tỉnh, Thành phố. Olympic 30-4. Đề thi và kiểm tra đội tuyển các cấp.

                                            12187821_1641976126063588_38896332569086

Câu 7 : 

Gọi giao của tiếp tuyến tại $A$ của $(AEM)$ với tiếp tuyến tại $B$ của $(BEN)$ là $G$. Dễ chứng minh $AG,BG$ cố định. Ta chứng minh $EF$ đi qua $G$ cố định. 

Thật vậy , gọi giao của tiếp tuyến tại $A$ của $(AEM)$ với $EF$ là $G'$, giao của tiếp tuyến tại $B$ của $(BEN)$ với $EF$ là $G''$. 

Ta có : $\angle ABG''=\angle ABE + \angle G''BE=\angle BNE + \angle ABE=\angle ACB+\angle DBC$ 

           $\angle BAG'=\angle BAE+\angle G'AE=\angle BAE+\angle AME=\angle ACB+\angle DBC$

Suy ra $G\equiv G' \equiv G''$ $(Q.E.D)$

Cách của em, anh xem đúng không :D

Tiếp tuyến tại $A$ của $(AEM)$ cắt tiếp tuyến tại $B$ của $(BEN)$ tại $I$. Ta có: 

$\widehat{IAB}=\widehat{IAC}+\widehat{BAC}=\widehat{AED}+\widehat{BAC}$

$\widehat{IBA}=\widehat{ABD}+\widehat{DBI}=\widehat{ABD}+\widehat{BEC}=\widehat{IAB}$

Do đó tam giác $IAB$ cân tại $I$ và điểm $I$ cố định (vì tam giác $IAB$ cân và góc ở đáy không đổi) nên $IA^2=IB^2$.

Mà $IA^2=IB^2$ nên $I$ thuộc trục đẳng phương của 2 đường tròn, $EF$ là trục đẳng phương của 2 đường tròn nên $EF$ đi qua $I$

=> $Q.E.D$

Hình gửi kèm

  • Untitled.png



#596469 Chứng minh M,N,I thẳng hàng

Đã gửi bởi Hoang Nhat Tuan on 01-11-2015 - 18:15 trong Hình học

Cho tứ giác ABCD nội tiếp đường tròn (O), AD cắt BC tại E, AC cắt BD tại F. Gọi M, N lần lượt là trung điểm của AB,CD, I là trung điểm của EF. Chứng minh M,N,I thẳng hàng.

Đây là đường thẳng Gauss, đã chứng minh tại Đây




#595828 $ab+bc+ca=3$. CMR: $\frac{1}{1+a^2}+\frac{1}{1+b^2}+...

Đã gửi bởi Hoang Nhat Tuan on 28-10-2015 - 21:33 trong Bất đẳng thức và cực trị

Cho $a,b,c\geqslant 0$ thỏa $ab+bc+ca=3$. CMR: $\frac{1}{1+a^2}+\frac{1}{1+b^2}+\frac{1}{1+c^2}\geqslant \frac{3}{2}$

Một cách không cần quy đồng (Tại 1 thanh niên nhác quy đồng nhờ ).

Vì $ab+bc+ca=3$ nên ta có thể giả sử $ab\geq 1$

Khi đó ta thu được BĐT sau:

$P=\frac{1}{1+a^2}+\frac{1}{1+b^2}+\frac{1}{1+c^2}\geq \frac{2}{1+ab}+\frac{1}{1+c^2}$

Do đó chỉ cần chứng minh:

$\frac{2}{1+ab}+\frac{1}{1+c^2}\geq \frac{3}{2}$

Chuyển vế rồi ta cần chứng minh: $c^2+3\geq ab+3abc^2$<=> $c^2+ac+bc\geq 3abc^2$ (dễ dàng chứng minh).

Dấu bằng như cách trên.




#595822 $ab+bc+ca=3$. CMR: $\frac{1}{1+a^2}+\frac{1}{1+b^2}+...

Đã gửi bởi Hoang Nhat Tuan on 28-10-2015 - 21:20 trong Bất đẳng thức và cực trị

Cho $a,b,c\geqslant 0$ thỏa $ab+bc+ca=3$. CMR: $\frac{1}{1+a^2}+\frac{1}{1+b^2}+\frac{1}{1+c^2}\geqslant \frac{3}{2}$

Sau khi quy đồng ta thu được BĐT mới:

$a^2+b^2+c^2+3\geq a^2b^2+b^2c^2+c^2a^2+3a^2b^2c^2$

Theo BĐT AM-GM thì: $abc\leq 1\leq \frac{a+b+c}{3}$ nên cần chứng minh:

$a^2+b^2+c^2+3\geq a^2b^2+b^2c^2+c^2a^2+abc(a+b+c)$

Ta làm sao để đưa BĐT về dạng đồng bậc thì tốt, muốn vậy ta làm như sau:

$(ab+bc+ca)(a^2+b^2+c^2)+(ab+bc+ca)^2\geq 3(a^2b^2+b^2c^2+c^2a^2)+3abc(a+b+c)$

Rút gọn BĐT trên ta được:

$\sum ab(a^2+b^2)\geq 2\sum a^2b^2$

Đây là 1 bất đẳng thức hiển nhiên

Dấu bằng xảy ra tại $a=b=c$ hoặc $a=b=\sqrt{3};c=0$ và các hoán vị




#595401 Đề thi chọn đội tuyển thi VMO tỉnh Thái Nguyên (2015 - 2016)

Đã gửi bởi Hoang Nhat Tuan on 26-10-2015 - 00:11 trong Thi HSG cấp Tỉnh, Thành phố. Olympic 30-4. Đề thi và kiểm tra đội tuyển các cấp.

Lời giải câu hình (khá dễ) :

Ta có: $DP^2=DO^2-R^2=OI^2+ID^2-(OE^2-EB^2)=OI^2+ID^2-OE^2+AI^2+IE^2=DA^2$

=> $DP=DA=DQ$

Dễ thấy $O,Q,I,D,P$ cùng thuộc 1 đường tròn.

Từ đó suy ra $\widehat{DIQ}=180^{\circ}-\widehat{DPQ}=180^{\circ}-\widehat{DQP}=\widehat{MQD}$

Do đó $\Delta DIQ\sim \Delta DQM=>\frac{DI}{DQ}=\frac{DQ}{DM}=>DI.DM=DQ^2=DA^2$

=> ĐPCM

Hình gửi kèm

  • Untitled.png



#595384 Đề thi chọn đội tuyển thi VMO tỉnh Thái Nguyên (2015 - 2016)

Đã gửi bởi Hoang Nhat Tuan on 25-10-2015 - 22:49 trong Thi HSG cấp Tỉnh, Thành phố. Olympic 30-4. Đề thi và kiểm tra đội tuyển các cấp.

KỲ THI LẬP ĐỘI TUYỂN DỰ THI HỌC SINH GIỎI QUỐC GIA LỚP 12 THPT NĂM HỌC 2015 - 2016

Ngày thi: 24/10/2015

 

Bài 1 (4 điểm). Cho các số thực dương $a$, $b$, $c$ thỏa mãn $abc=1$. Chứng minh rằng

\[a^4+b^4+c^4+a+b+c+\dfrac{2a}{b^2+c^2}+\dfrac{2b}{c^2+a^2}+\dfrac{2c}{a^2+b^2}\geqslant 9\]

 

Ta có BĐT sau: $x^4+y^4\geq xy(x^2+y^2)$

Áp dụng:

$\sum (a^4+b^4)+2(a+b+c)+\sum \frac{4b}{c^2+a^2}\geq \sum ab(a^2+b^2)+\sum \frac{4c}{a^2+b^2}+2(a+b+c)\geq 12\sqrt{abc}+6.\sqrt[3]{abc}=18$

=> ĐPCM




#595374 Mới chế bài BĐT : $27abc \geq 15(ab+ac+bc) - 4$

Đã gửi bởi Hoang Nhat Tuan on 25-10-2015 - 22:08 trong Bất đẳng thức và cực trị

Bất đẳng thức sau do mình chế ra, được đánh giá là rất mạnh, mong cao thủ chỉ giáo ạ 

 

Cho 3 số dương a,b,c thỏa :  a+b+c = 1

 

cmr :

$27abc \geq 15(ab+ac+bc) - 4$

Đặt $p=a+b+c;q=ab+bc+ca;r=abc$

BĐT Schur bậc 3:

$r\geq\frac{p(4q-p)}{9}=>27r\geq 3(4q-1)$

Nghĩa là chỉ cần chứng minh: $3(4q-1)\geq 15q-4<=>1\geq 3q<=>p^2\geq 3q$

BĐT yếu hơn cả Schur mà bạn bảo mạnh thì cũng hài nhỉ :D




#594629 $\left\{\begin{matrix} x,y,z\geq 0...

Đã gửi bởi Hoang Nhat Tuan on 20-10-2015 - 21:46 trong Phương trình - Hệ phương trình - Bất phương trình

TST 2001 cha, hình như đề là:

Cho x,y,z>0 thỏa mãn $2x+4y+7z\leq 2xyz$. Tìm giá trị nhỏ nhất của biểu thức $x+y+z$

(Công nhận mấy ông ác thật, thời gian thì có tí, nhẩm điểm rơi sao ra được, ai ngờ KQ là $7,5$, quá đắng @@ )




#594602 $$\sum \frac{1}{4a^2+b^2+c^2}\le...

Đã gửi bởi Hoang Nhat Tuan on 20-10-2015 - 19:35 trong Bất đẳng thức và cực trị

Cho $a,b,c$ không âm và không có hai số nào đồng thời bằng $0$. Chứng minh:

$$\sum \frac{1}{4a^2+b^2+c^2}\leq \frac{1}{2\sum a^2}+\frac{1}{\sum ab}$$

$(Vasc)$                                          

Ta có: $\sum \frac{(a+b+c)^2}{4a^2+b^2+c^2}\leq \sum \frac{a^2}{2a^2}+\sum \frac{b^2}{a^2+b^2}+\sum \frac{c^2}{a^2+c^2}=\frac{9}{2}$

Và: $\frac{(a+b+c)^2}{2\sum a^2}+\frac{4(a+b+c)^2}{4\sum ab}\geq \frac{9(a+b+c)^2}{2(a+b+c)^2}=\frac{9}{2}$

Từ đó => ĐPCM

Cho $a,b,c$ không âm thỏa mãn $a+b+c=3$.Chứng minh rằng:

 

$$\frac{1}{4a^2+b^2+c^2}+\frac{1}{4b^2+a^2+c^2}+\frac{1}{4c^2+a^2+b^2}\leq \frac{1}{2}$$

Bài của anh giải như trên @@




#594243 $\frac{x^{2}y}{z}+\frac{y^...

Đã gửi bởi Hoang Nhat Tuan on 18-10-2015 - 12:55 trong Bất đẳng thức - Cực trị

Cho các số $x\geq y\geq z>0$. Chứng minh rằng:
$\frac{x^{2}y}{z}+\frac{y^{2}z}{x}+\frac{z^{2}x}{y}\geq x^{2}+y^{2}+z^{2}$

Sử dụng BĐT C-S thì:

$\sqrt{(\frac{x^2y}{z}+\frac{y^2z}{x}+\frac{z^2x}{y}).(\frac{x^2z}{y}+\frac{y^2x}{z}+\frac{z^2y}{x})}\geq x^2+y^2+z^2$

Lại có:

$\sum \frac{x^2y}{z}\geq \sum \frac{x^2z}{y}$ (nhờ giả thiết)

Từ đó => ĐPCM




#592789 Chứng minh rằng có thể chọn ra $7$ đứa con gái sao cho mỗi thằng co...

Đã gửi bởi Hoang Nhat Tuan on 08-10-2015 - 22:07 trong Tổ hợp - Xác suất và thống kê - Số phức

Cho $100$ thằng con trai và $25$ đứa con gái, biết rằng mỗi thằng con trai thích ít nhất $10$ đứa con gái trong số đó. Chứng minh rằng có thể chọn ra $7$ đứa con gái sao cho mỗi thằng con trai thích ít nhất $1$ trong $7$ đứa con gái đó :D




#592584 $P=\frac{1}{x^{2}+z^{2}}+\frac{1}{y^{2}+z^{2}}+\sqrt{x+y+...

Đã gửi bởi Hoang Nhat Tuan on 07-10-2015 - 19:58 trong Bất đẳng thức và cực trị

cho $x,y>0, z\geq 0$, z là số nhỏ nhất trong 3 số. Tìm giá trị của $P=\frac{1}{x^{2}+z^{2}}+\frac{1}{y^{2}+z^{2}}+\sqrt{x+y+z}$

$z$ nhỏ nhất trong 3 số nên ta có các đánh giá sau:

$x^2+z^2\leq (x+\frac{z}{2})^2$

và $y^2+z^2\leq (y+\frac{z}{2})^2$

Do đó: $P\geq \frac{1}{(x+\frac{z}{2})^2}+\frac{1}{(y+\frac{z}{2})^2}+\sqrt{x+y+z}\geq \frac{8}{(x+y+z)^2}+\sqrt{x+y+z}$

$=\frac{8}{(x+y+z)^2}+4.\frac{\sqrt{x+y+z}}{4}\geq \frac{5}{2}$

Dấu bằng xảy ra tại $x=y=2;z=0$




#592109 Đề thi chọn đội tuyển lần 2 trường THPT chuyên Hưng Yên

Đã gửi bởi Hoang Nhat Tuan on 04-10-2015 - 20:57 trong Thi HSG cấp Tỉnh, Thành phố. Olympic 30-4. Đề thi và kiểm tra đội tuyển các cấp.

 

Đề thi chọn đội tuyển lần 2 trường THPT chuyên Hưng Yên

 

Câu6 Trong một hội nghị có 100 người.Trong đó có 15 người Pháp,mỗi người quen với ít nhất 70 đại biểu và 85 người Đức,mỗi người quen với không quá 10 đại biểu.Họ được phân vào 21 phòng.Chứng minh rằng có một phòng nào đó không chứa một cặp nào quen nhau.

 

Vì mỗi người Pháp quen ít nhất 70 đại biểu nên mỗi người pháp quen ít nhất $70-14=56$ người Đức.

Do đó có 15 người Pháp thì số cặp người Pháp quen người Đức là 840 cặp.

Gọi n là số người Đức quen $\leq 9$ người Pháp thì ta có:

$840\leq (85-n).10+9n$ nên $n \leq 10$

Có tổng cộng 21 phòng, mà có 15 người Pháp nên có 6 phòng có toàn người Đức.

Vì số người Đức quen nhau $\leq 10$ nên trong 6 phòng này ít nhất có 1 phòng chỉ chứa 1 người Đức ( trong n người Đức) hoặc không có người Đức nào trong n người Đức đó.

Vì $85-n$ người Đức còn lại không quen biết nhau nên từ đó ta suy ra ĐPCM




#592103 Đề thi chọn đội tuyển lần 2 trường THPT chuyên Hưng Yên

Đã gửi bởi Hoang Nhat Tuan on 04-10-2015 - 20:44 trong Thi HSG cấp Tỉnh, Thành phố. Olympic 30-4. Đề thi và kiểm tra đội tuyển các cấp.

Ta có: $x^{3}< 2^{y}< (x+2)^{3}=>2^{y}=(x+1)^{3}$

Khai triển =>x =>y

Căn cứ vào đâu mà từ cái $x^3<2^y<(x+2)^3$ suy ra cái sau được bạn, đề bài có nói $2^y$ là lập phương của một số tự nhiên đâu.

Lời giải hoàn toàn sai lầm bạn nhé :)




#592080 Đề thi chọn đội tuyển lần 2 trường THPT chuyên Hưng Yên

Đã gửi bởi Hoang Nhat Tuan on 04-10-2015 - 19:52 trong Thi HSG cấp Tỉnh, Thành phố. Olympic 30-4. Đề thi và kiểm tra đội tuyển các cấp.

 

Đề thi chọn đội tuyển lần 2 trường THPT chuyên Hưng Yên

 

Câu2 Tìm số tự nhiên x,y thỏa mãn $2^y=1+x+x^2+x^3$

Dễ thấy với $y=0$ thì $x=0$

Ta có: $2^y=(x^2+1)(x+1)$

Đặt $x^2+1=2^a;x+1=2^b$ và $a+b=y$, $a \geq b$ thì:

$x^2-x=2^a-2^b$

=>$x(x-1)=2^b(2^{a-b}-1)$

Với $x=1$ thì $a=b$ và $y=1$, trường hợp x khác 1 thì

$(x;x-1)=1$ do đó có các trường hợp sau:

$x=2^b; x-1=2^{a-b}-1$ (1)

$x=2^{a-b}-1;x-1=2^b$  (2)

Mà $x+1=2^b$ nên cả 2 TH đều không có nghiệm

Ngoài ra còn 1 TH là x=2 nữa nhưng thay vào vô lý 




#592014 $P=\sum \frac{a^{2}}{b^{2}+...

Đã gửi bởi Hoang Nhat Tuan on 04-10-2015 - 12:12 trong Bất đẳng thức - Cực trị

Cho $\left\{\begin{matrix} a,b,c>0 & \\ a^{2}+b^{2}+c^{2}=3 & \end{matrix}\right.$

Tìm min của: $P=\sum \frac{a^{2}}{b^{2}+c^{2}+bc}$

Cái điều kiện không liên quan lắm

$P=\sum \frac{a^2}{b^2+c^2+bc}\geq \sum \frac{2a^2}{3(b^2+c^2)}\geq 1$ ( BĐT Nesbit)




#591924 Chứng minh $\sum \frac{x+1}{y+1}\leq...

Đã gửi bởi Hoang Nhat Tuan on 03-10-2015 - 22:25 trong Bất đẳng thức và cực trị

Bạn tham khảo ở đây nhé

http://diendantoanho...leq-sum-fracxy/

Cách giải đó hoàn toàn sai lầm.

 

Cho $x,y,z>0$   .  Chứng minh $\sum \frac{x+1}{y+1}\leq \sum \frac{x}{y}$

Đối với bài này thì e có 1 cách trâu bò và phương pháp đúng là "trâu bò"

Sau khi quy đồng thì ta cần chứng minh:

$\sum x^3z^2+\sum x^2y^2+\sum x^3z+\sum x^2z\geq \sum x^2yz^2+2\sum x^2yz+3xyz$

Chứng minh BĐT trên dựa vào 4 BĐT sau:

$\sum x^3z^2\geq \sum x^2yz^2$

$\sum x^2y^2\geq \sum x^2yz$

$\sum x^3z \geq \sum x^2yz$

$\sum x^2z \geq 3xyz$

Các BĐT trên dễ dàng chứng minh bằng AM-GM và C-S